Lattice Paths that Avoid a Point











up vote
9
down vote

favorite
1












My house is located at $(0,0)$ and the supermarket is located at $(7,5)$. I can only move in the upwards or in rightward directions. How many different routes are there? Obviously, ${12}choose{7}$ $=$ ${12}choose{5}$ $=$ $792$. How many paths are there if I want to avoid the really busy intersection located at $(3,3)$?



I have three ideas, all of which lead to different answers:



1) My first idea was to label out the grid and write the number at each intersection that represents how many paths cross that point. This was equivalent to writing out pascals triangle in a tilted diagonal way, until I reached $(3,3)$ whereI had to write a $0$, but then I proceeded as you'd expect, the intersection's number was equal to the one below it added to the one to the left of it. At $(7,5)$ I ended up getting $490$.



After this, I wanted a combinatorial way to confirm my answer, so I tried:



2)counting all the paths from $(0,0)$ to $(3,3)$ and subtracting those away from $792$. So here I would subtract ${6}choose{3}$ $=20$.



3)counting all the paths from $(3,3)$ to $(7,5)$ and subtracting those away from $792$. So here I would subtract ${6}choose{4}$ $=15$.



So my question is: which method of thinking is correct and why do the other two not lead to the correct answer (what do they over/under count)? I suspect that my first try was correct, and the answer is $490$, but I don't see the algorithmic way of seeing it...



EDIT: Made a correction to my grid. I wrote a $34$ instead of a $36$ for the intersection at $(7,2)$. Woops!










share|cite|improve this question




















  • 3




    I would have expected your supermarket to be at $(7,11)$... :P
    – Asaf Karagila
    yesterday















up vote
9
down vote

favorite
1












My house is located at $(0,0)$ and the supermarket is located at $(7,5)$. I can only move in the upwards or in rightward directions. How many different routes are there? Obviously, ${12}choose{7}$ $=$ ${12}choose{5}$ $=$ $792$. How many paths are there if I want to avoid the really busy intersection located at $(3,3)$?



I have three ideas, all of which lead to different answers:



1) My first idea was to label out the grid and write the number at each intersection that represents how many paths cross that point. This was equivalent to writing out pascals triangle in a tilted diagonal way, until I reached $(3,3)$ whereI had to write a $0$, but then I proceeded as you'd expect, the intersection's number was equal to the one below it added to the one to the left of it. At $(7,5)$ I ended up getting $490$.



After this, I wanted a combinatorial way to confirm my answer, so I tried:



2)counting all the paths from $(0,0)$ to $(3,3)$ and subtracting those away from $792$. So here I would subtract ${6}choose{3}$ $=20$.



3)counting all the paths from $(3,3)$ to $(7,5)$ and subtracting those away from $792$. So here I would subtract ${6}choose{4}$ $=15$.



So my question is: which method of thinking is correct and why do the other two not lead to the correct answer (what do they over/under count)? I suspect that my first try was correct, and the answer is $490$, but I don't see the algorithmic way of seeing it...



EDIT: Made a correction to my grid. I wrote a $34$ instead of a $36$ for the intersection at $(7,2)$. Woops!










share|cite|improve this question




















  • 3




    I would have expected your supermarket to be at $(7,11)$... :P
    – Asaf Karagila
    yesterday













up vote
9
down vote

favorite
1









up vote
9
down vote

favorite
1






1





My house is located at $(0,0)$ and the supermarket is located at $(7,5)$. I can only move in the upwards or in rightward directions. How many different routes are there? Obviously, ${12}choose{7}$ $=$ ${12}choose{5}$ $=$ $792$. How many paths are there if I want to avoid the really busy intersection located at $(3,3)$?



I have three ideas, all of which lead to different answers:



1) My first idea was to label out the grid and write the number at each intersection that represents how many paths cross that point. This was equivalent to writing out pascals triangle in a tilted diagonal way, until I reached $(3,3)$ whereI had to write a $0$, but then I proceeded as you'd expect, the intersection's number was equal to the one below it added to the one to the left of it. At $(7,5)$ I ended up getting $490$.



After this, I wanted a combinatorial way to confirm my answer, so I tried:



2)counting all the paths from $(0,0)$ to $(3,3)$ and subtracting those away from $792$. So here I would subtract ${6}choose{3}$ $=20$.



3)counting all the paths from $(3,3)$ to $(7,5)$ and subtracting those away from $792$. So here I would subtract ${6}choose{4}$ $=15$.



So my question is: which method of thinking is correct and why do the other two not lead to the correct answer (what do they over/under count)? I suspect that my first try was correct, and the answer is $490$, but I don't see the algorithmic way of seeing it...



EDIT: Made a correction to my grid. I wrote a $34$ instead of a $36$ for the intersection at $(7,2)$. Woops!










share|cite|improve this question















My house is located at $(0,0)$ and the supermarket is located at $(7,5)$. I can only move in the upwards or in rightward directions. How many different routes are there? Obviously, ${12}choose{7}$ $=$ ${12}choose{5}$ $=$ $792$. How many paths are there if I want to avoid the really busy intersection located at $(3,3)$?



I have three ideas, all of which lead to different answers:



1) My first idea was to label out the grid and write the number at each intersection that represents how many paths cross that point. This was equivalent to writing out pascals triangle in a tilted diagonal way, until I reached $(3,3)$ whereI had to write a $0$, but then I proceeded as you'd expect, the intersection's number was equal to the one below it added to the one to the left of it. At $(7,5)$ I ended up getting $490$.



After this, I wanted a combinatorial way to confirm my answer, so I tried:



2)counting all the paths from $(0,0)$ to $(3,3)$ and subtracting those away from $792$. So here I would subtract ${6}choose{3}$ $=20$.



3)counting all the paths from $(3,3)$ to $(7,5)$ and subtracting those away from $792$. So here I would subtract ${6}choose{4}$ $=15$.



So my question is: which method of thinking is correct and why do the other two not lead to the correct answer (what do they over/under count)? I suspect that my first try was correct, and the answer is $490$, but I don't see the algorithmic way of seeing it...



EDIT: Made a correction to my grid. I wrote a $34$ instead of a $36$ for the intersection at $(7,2)$. Woops!







combinatorics






share|cite|improve this question















share|cite|improve this question













share|cite|improve this question




share|cite|improve this question








edited 2 days ago

























asked 2 days ago









ruferd

22318




22318








  • 3




    I would have expected your supermarket to be at $(7,11)$... :P
    – Asaf Karagila
    yesterday














  • 3




    I would have expected your supermarket to be at $(7,11)$... :P
    – Asaf Karagila
    yesterday








3




3




I would have expected your supermarket to be at $(7,11)$... :P
– Asaf Karagila
yesterday




I would have expected your supermarket to be at $(7,11)$... :P
– Asaf Karagila
yesterday










2 Answers
2






active

oldest

votes

















up vote
14
down vote



accepted










You need to look at the complement, which is all the paths from $(0,0)$ to $(7,5)$ that pass through $(3,3)$.



As you noted, the number of paths from $(0,0)$ to $(3,3)$ is ${6choose3} = 20$, and the number of paths from $(3,3)$ to $(7,5)$ is ${6choose4} = 15$. So the number of paths from $(0,0)$ to $(7,5)$ that pass through $(3,3)$ is all combinations of the above paths, which means $20cdot15=300$ paths.



$792-300=492$, and so $492$ is the final answer.






share|cite|improve this answer








New contributor




AlephZero is a new contributor to this site. Take care in asking for clarification, commenting, and answering.
Check out our Code of Conduct.


















  • Ok, I see now...I wasn't accounting for each new branch that I could take from $(3,3)$ to $(7,5)$ after I had already gone from $(0,0)$ to $(3,3)$. I guess I should have reflected on my results of option 2 and 3 and seen if I could use ${6}choose{4}$ and ${6}choose{3}$ in some way with 792 to get the answer. The only reasonable way would be to multiply them together and subtract, but this is the argument to back that idea up, thank you! And also, it appears that my grid in option 1 was off somehow! Woops!
    – ruferd
    2 days ago


















up vote
4
down vote













Your idea to subtract the number of paths passing through $(3,3)$ from $792$ was a good idea, but you didn't carry it out quite right. The number of paths that pass through $(3,3)$ is not $binom{6}{3}$ nor $binom{6}{4}$, but $binom{6}{3}binom{6}{4}$. This is because every path from $(0,0)$ to $(7,5)$ through $(3,3)$ consists of two "mini-paths," one from $(0,0)$ to $(3,3)$ and one from $(3,3)$ to $(7,5)$. There are $binom{6}{3}$ ways to choose the first path and $binom{6}{4}$ ways to choose the second, resulting in $binom{6}{3}binom{6}{4}$ ways to choose the entire path, and $792-binom{6}{3}binom{6}{4}$ paths avoiding $(3,3)$.






share|cite|improve this answer





















  • AH ok, thank you. a path from $(0,0)$ to $(3,3)$ would also have ${6}choose{4}$ ways to get to $(7,5)$, so I'd multiply, thanks, it is so obvious now!
    – ruferd
    2 days ago











Your Answer





StackExchange.ifUsing("editor", function () {
return StackExchange.using("mathjaxEditing", function () {
StackExchange.MarkdownEditor.creationCallbacks.add(function (editor, postfix) {
StackExchange.mathjaxEditing.prepareWmdForMathJax(editor, postfix, [["$", "$"], ["\\(","\\)"]]);
});
});
}, "mathjax-editing");

StackExchange.ready(function() {
var channelOptions = {
tags: "".split(" "),
id: "69"
};
initTagRenderer("".split(" "), "".split(" "), channelOptions);

StackExchange.using("externalEditor", function() {
// Have to fire editor after snippets, if snippets enabled
if (StackExchange.settings.snippets.snippetsEnabled) {
StackExchange.using("snippets", function() {
createEditor();
});
}
else {
createEditor();
}
});

function createEditor() {
StackExchange.prepareEditor({
heartbeatType: 'answer',
convertImagesToLinks: true,
noModals: true,
showLowRepImageUploadWarning: true,
reputationToPostImages: 10,
bindNavPrevention: true,
postfix: "",
imageUploader: {
brandingHtml: "Powered by u003ca class="icon-imgur-white" href="https://imgur.com/"u003eu003c/au003e",
contentPolicyHtml: "User contributions licensed under u003ca href="https://creativecommons.org/licenses/by-sa/3.0/"u003ecc by-sa 3.0 with attribution requiredu003c/au003e u003ca href="https://stackoverflow.com/legal/content-policy"u003e(content policy)u003c/au003e",
allowUrls: true
},
noCode: true, onDemand: true,
discardSelector: ".discard-answer"
,immediatelyShowMarkdownHelp:true
});


}
});














draft saved

draft discarded


















StackExchange.ready(
function () {
StackExchange.openid.initPostLogin('.new-post-login', 'https%3a%2f%2fmath.stackexchange.com%2fquestions%2f3020627%2flattice-paths-that-avoid-a-point%23new-answer', 'question_page');
}
);

Post as a guest















Required, but never shown

























2 Answers
2






active

oldest

votes








2 Answers
2






active

oldest

votes









active

oldest

votes






active

oldest

votes








up vote
14
down vote



accepted










You need to look at the complement, which is all the paths from $(0,0)$ to $(7,5)$ that pass through $(3,3)$.



As you noted, the number of paths from $(0,0)$ to $(3,3)$ is ${6choose3} = 20$, and the number of paths from $(3,3)$ to $(7,5)$ is ${6choose4} = 15$. So the number of paths from $(0,0)$ to $(7,5)$ that pass through $(3,3)$ is all combinations of the above paths, which means $20cdot15=300$ paths.



$792-300=492$, and so $492$ is the final answer.






share|cite|improve this answer








New contributor




AlephZero is a new contributor to this site. Take care in asking for clarification, commenting, and answering.
Check out our Code of Conduct.


















  • Ok, I see now...I wasn't accounting for each new branch that I could take from $(3,3)$ to $(7,5)$ after I had already gone from $(0,0)$ to $(3,3)$. I guess I should have reflected on my results of option 2 and 3 and seen if I could use ${6}choose{4}$ and ${6}choose{3}$ in some way with 792 to get the answer. The only reasonable way would be to multiply them together and subtract, but this is the argument to back that idea up, thank you! And also, it appears that my grid in option 1 was off somehow! Woops!
    – ruferd
    2 days ago















up vote
14
down vote



accepted










You need to look at the complement, which is all the paths from $(0,0)$ to $(7,5)$ that pass through $(3,3)$.



As you noted, the number of paths from $(0,0)$ to $(3,3)$ is ${6choose3} = 20$, and the number of paths from $(3,3)$ to $(7,5)$ is ${6choose4} = 15$. So the number of paths from $(0,0)$ to $(7,5)$ that pass through $(3,3)$ is all combinations of the above paths, which means $20cdot15=300$ paths.



$792-300=492$, and so $492$ is the final answer.






share|cite|improve this answer








New contributor




AlephZero is a new contributor to this site. Take care in asking for clarification, commenting, and answering.
Check out our Code of Conduct.


















  • Ok, I see now...I wasn't accounting for each new branch that I could take from $(3,3)$ to $(7,5)$ after I had already gone from $(0,0)$ to $(3,3)$. I guess I should have reflected on my results of option 2 and 3 and seen if I could use ${6}choose{4}$ and ${6}choose{3}$ in some way with 792 to get the answer. The only reasonable way would be to multiply them together and subtract, but this is the argument to back that idea up, thank you! And also, it appears that my grid in option 1 was off somehow! Woops!
    – ruferd
    2 days ago













up vote
14
down vote



accepted







up vote
14
down vote



accepted






You need to look at the complement, which is all the paths from $(0,0)$ to $(7,5)$ that pass through $(3,3)$.



As you noted, the number of paths from $(0,0)$ to $(3,3)$ is ${6choose3} = 20$, and the number of paths from $(3,3)$ to $(7,5)$ is ${6choose4} = 15$. So the number of paths from $(0,0)$ to $(7,5)$ that pass through $(3,3)$ is all combinations of the above paths, which means $20cdot15=300$ paths.



$792-300=492$, and so $492$ is the final answer.






share|cite|improve this answer








New contributor




AlephZero is a new contributor to this site. Take care in asking for clarification, commenting, and answering.
Check out our Code of Conduct.









You need to look at the complement, which is all the paths from $(0,0)$ to $(7,5)$ that pass through $(3,3)$.



As you noted, the number of paths from $(0,0)$ to $(3,3)$ is ${6choose3} = 20$, and the number of paths from $(3,3)$ to $(7,5)$ is ${6choose4} = 15$. So the number of paths from $(0,0)$ to $(7,5)$ that pass through $(3,3)$ is all combinations of the above paths, which means $20cdot15=300$ paths.



$792-300=492$, and so $492$ is the final answer.







share|cite|improve this answer








New contributor




AlephZero is a new contributor to this site. Take care in asking for clarification, commenting, and answering.
Check out our Code of Conduct.









share|cite|improve this answer



share|cite|improve this answer






New contributor




AlephZero is a new contributor to this site. Take care in asking for clarification, commenting, and answering.
Check out our Code of Conduct.









answered 2 days ago









AlephZero

25116




25116




New contributor




AlephZero is a new contributor to this site. Take care in asking for clarification, commenting, and answering.
Check out our Code of Conduct.





New contributor





AlephZero is a new contributor to this site. Take care in asking for clarification, commenting, and answering.
Check out our Code of Conduct.






AlephZero is a new contributor to this site. Take care in asking for clarification, commenting, and answering.
Check out our Code of Conduct.












  • Ok, I see now...I wasn't accounting for each new branch that I could take from $(3,3)$ to $(7,5)$ after I had already gone from $(0,0)$ to $(3,3)$. I guess I should have reflected on my results of option 2 and 3 and seen if I could use ${6}choose{4}$ and ${6}choose{3}$ in some way with 792 to get the answer. The only reasonable way would be to multiply them together and subtract, but this is the argument to back that idea up, thank you! And also, it appears that my grid in option 1 was off somehow! Woops!
    – ruferd
    2 days ago


















  • Ok, I see now...I wasn't accounting for each new branch that I could take from $(3,3)$ to $(7,5)$ after I had already gone from $(0,0)$ to $(3,3)$. I guess I should have reflected on my results of option 2 and 3 and seen if I could use ${6}choose{4}$ and ${6}choose{3}$ in some way with 792 to get the answer. The only reasonable way would be to multiply them together and subtract, but this is the argument to back that idea up, thank you! And also, it appears that my grid in option 1 was off somehow! Woops!
    – ruferd
    2 days ago
















Ok, I see now...I wasn't accounting for each new branch that I could take from $(3,3)$ to $(7,5)$ after I had already gone from $(0,0)$ to $(3,3)$. I guess I should have reflected on my results of option 2 and 3 and seen if I could use ${6}choose{4}$ and ${6}choose{3}$ in some way with 792 to get the answer. The only reasonable way would be to multiply them together and subtract, but this is the argument to back that idea up, thank you! And also, it appears that my grid in option 1 was off somehow! Woops!
– ruferd
2 days ago




Ok, I see now...I wasn't accounting for each new branch that I could take from $(3,3)$ to $(7,5)$ after I had already gone from $(0,0)$ to $(3,3)$. I guess I should have reflected on my results of option 2 and 3 and seen if I could use ${6}choose{4}$ and ${6}choose{3}$ in some way with 792 to get the answer. The only reasonable way would be to multiply them together and subtract, but this is the argument to back that idea up, thank you! And also, it appears that my grid in option 1 was off somehow! Woops!
– ruferd
2 days ago










up vote
4
down vote













Your idea to subtract the number of paths passing through $(3,3)$ from $792$ was a good idea, but you didn't carry it out quite right. The number of paths that pass through $(3,3)$ is not $binom{6}{3}$ nor $binom{6}{4}$, but $binom{6}{3}binom{6}{4}$. This is because every path from $(0,0)$ to $(7,5)$ through $(3,3)$ consists of two "mini-paths," one from $(0,0)$ to $(3,3)$ and one from $(3,3)$ to $(7,5)$. There are $binom{6}{3}$ ways to choose the first path and $binom{6}{4}$ ways to choose the second, resulting in $binom{6}{3}binom{6}{4}$ ways to choose the entire path, and $792-binom{6}{3}binom{6}{4}$ paths avoiding $(3,3)$.






share|cite|improve this answer





















  • AH ok, thank you. a path from $(0,0)$ to $(3,3)$ would also have ${6}choose{4}$ ways to get to $(7,5)$, so I'd multiply, thanks, it is so obvious now!
    – ruferd
    2 days ago















up vote
4
down vote













Your idea to subtract the number of paths passing through $(3,3)$ from $792$ was a good idea, but you didn't carry it out quite right. The number of paths that pass through $(3,3)$ is not $binom{6}{3}$ nor $binom{6}{4}$, but $binom{6}{3}binom{6}{4}$. This is because every path from $(0,0)$ to $(7,5)$ through $(3,3)$ consists of two "mini-paths," one from $(0,0)$ to $(3,3)$ and one from $(3,3)$ to $(7,5)$. There are $binom{6}{3}$ ways to choose the first path and $binom{6}{4}$ ways to choose the second, resulting in $binom{6}{3}binom{6}{4}$ ways to choose the entire path, and $792-binom{6}{3}binom{6}{4}$ paths avoiding $(3,3)$.






share|cite|improve this answer





















  • AH ok, thank you. a path from $(0,0)$ to $(3,3)$ would also have ${6}choose{4}$ ways to get to $(7,5)$, so I'd multiply, thanks, it is so obvious now!
    – ruferd
    2 days ago













up vote
4
down vote










up vote
4
down vote









Your idea to subtract the number of paths passing through $(3,3)$ from $792$ was a good idea, but you didn't carry it out quite right. The number of paths that pass through $(3,3)$ is not $binom{6}{3}$ nor $binom{6}{4}$, but $binom{6}{3}binom{6}{4}$. This is because every path from $(0,0)$ to $(7,5)$ through $(3,3)$ consists of two "mini-paths," one from $(0,0)$ to $(3,3)$ and one from $(3,3)$ to $(7,5)$. There are $binom{6}{3}$ ways to choose the first path and $binom{6}{4}$ ways to choose the second, resulting in $binom{6}{3}binom{6}{4}$ ways to choose the entire path, and $792-binom{6}{3}binom{6}{4}$ paths avoiding $(3,3)$.






share|cite|improve this answer












Your idea to subtract the number of paths passing through $(3,3)$ from $792$ was a good idea, but you didn't carry it out quite right. The number of paths that pass through $(3,3)$ is not $binom{6}{3}$ nor $binom{6}{4}$, but $binom{6}{3}binom{6}{4}$. This is because every path from $(0,0)$ to $(7,5)$ through $(3,3)$ consists of two "mini-paths," one from $(0,0)$ to $(3,3)$ and one from $(3,3)$ to $(7,5)$. There are $binom{6}{3}$ ways to choose the first path and $binom{6}{4}$ ways to choose the second, resulting in $binom{6}{3}binom{6}{4}$ ways to choose the entire path, and $792-binom{6}{3}binom{6}{4}$ paths avoiding $(3,3)$.







share|cite|improve this answer












share|cite|improve this answer



share|cite|improve this answer










answered 2 days ago









Frpzzd

20.4k638104




20.4k638104












  • AH ok, thank you. a path from $(0,0)$ to $(3,3)$ would also have ${6}choose{4}$ ways to get to $(7,5)$, so I'd multiply, thanks, it is so obvious now!
    – ruferd
    2 days ago


















  • AH ok, thank you. a path from $(0,0)$ to $(3,3)$ would also have ${6}choose{4}$ ways to get to $(7,5)$, so I'd multiply, thanks, it is so obvious now!
    – ruferd
    2 days ago
















AH ok, thank you. a path from $(0,0)$ to $(3,3)$ would also have ${6}choose{4}$ ways to get to $(7,5)$, so I'd multiply, thanks, it is so obvious now!
– ruferd
2 days ago




AH ok, thank you. a path from $(0,0)$ to $(3,3)$ would also have ${6}choose{4}$ ways to get to $(7,5)$, so I'd multiply, thanks, it is so obvious now!
– ruferd
2 days ago


















draft saved

draft discarded




















































Thanks for contributing an answer to Mathematics Stack Exchange!


  • Please be sure to answer the question. Provide details and share your research!

But avoid



  • Asking for help, clarification, or responding to other answers.

  • Making statements based on opinion; back them up with references or personal experience.


Use MathJax to format equations. MathJax reference.


To learn more, see our tips on writing great answers.





Some of your past answers have not been well-received, and you're in danger of being blocked from answering.


Please pay close attention to the following guidance:


  • Please be sure to answer the question. Provide details and share your research!

But avoid



  • Asking for help, clarification, or responding to other answers.

  • Making statements based on opinion; back them up with references or personal experience.


To learn more, see our tips on writing great answers.




draft saved


draft discarded














StackExchange.ready(
function () {
StackExchange.openid.initPostLogin('.new-post-login', 'https%3a%2f%2fmath.stackexchange.com%2fquestions%2f3020627%2flattice-paths-that-avoid-a-point%23new-answer', 'question_page');
}
);

Post as a guest















Required, but never shown





















































Required, but never shown














Required, but never shown












Required, but never shown







Required, but never shown

































Required, but never shown














Required, but never shown












Required, but never shown







Required, but never shown







Popular posts from this blog

Quarter-circle Tiles

build a pushdown automaton that recognizes the reverse language of a given pushdown automaton?

Mont Emei